Comprehension
It is sometimes mooted that there can be democracy in a two party system. That would be correct if politics were a game like cricket or football; but politics is not sports.
Question: 1

Which of the following would strengthen the argument?

Show Hint

When strengthening an argument, look for statements that reinforce the central concern or criticism expressed.
Updated On: Aug 7, 2025
Hide Solution
collegedunia
Verified By Collegedunia

Solution and Explanation

The passage criticizes the two-party political system by arguing that it is not ideal for democracy. It suggests that the comparison between politics and sports (e.g., cricket or football) is flawed because politics is not a game.
The key argument is that a two-party system may not provide enough choice to voters, unlike a game which inherently involves only two sides.
Option 1: Saying “two party system functions well” contradicts the argument. This would actually weaken the author’s stance by suggesting that a two-party system is beneficial.
Option 2: “Politics is a dirty game” neither strengthens nor weakens the argument directly. It is a vague, emotionally charged statement that does not address the central issue of voter choice or democratic ideals.
Option 3: This supports the argument directly. If two political parties limit the voter’s choice, then democracy is constrained in a two-party system. This strengthens the claim that politics is unlike sports and that a two-party system may be insufficient.
Option 4: Incorrect, because Option 3 strengthens the argument.
Hence, the correct answer is Option 3.
Was this answer helpful?
0
0
Question: 2

Which of the following would weaken the argument?

Show Hint

To weaken an argument, find a statement that supports the opposing viewpoint or negates the author's reasoning.
Updated On: Aug 7, 2025
Hide Solution
collegedunia
Verified By Collegedunia

Solution and Explanation

The argument in the passage draws a distinction between politics and sports. It claims that treating politics as a two-sided game (like football or cricket) is incorrect because politics is not a sport and therefore requires more complexity than just two teams (parties).
Option 1: This directly weakens the argument. If politics is like a game, similar to football, then the analogy used by proponents of the two-party system holds. This would support the idea that democracy can function with two parties. Hence, it contradicts the author’s main point and weakens the argument.
Option 2: This restates the author’s claim that “politics is not a sport” and therefore supports, not weakens, the argument.
Option 3: Political struggle for power is a neutral observation and does not affect the argument about the validity of the two-party model. It does not weaken the central claim.
Option 4: Incorrect, because Option 1 weakens the argument.
Hence, the correct answer is Option 1.
Was this answer helpful?
0
0
Question: 3

The assumption/assumptions of the argument is/are which of the following?
Statement I: Politics is not a game.
Statement II: Two party system is ideal for democracy.
Statement III: Cricket is played by two teams.

Show Hint

Assumptions are unstated but necessary beliefs. Facts and examples are not assumptions.
Updated On: Aug 7, 2025
Hide Solution
collegedunia
Verified By Collegedunia

Solution and Explanation

To evaluate assumptions, we must determine which statements are necessary for the argument to make sense.
The argument claims that applying the two-party system to democracy is flawed because politics is not a game.
Statement I: This is assumed directly. The entire argument rests on the idea that politics is fundamentally different from sports, and therefore should not be treated like a game. Without this assumption, the argument loses meaning.
Statement II: This is not assumed. The author argues against the idea that a two-party system is ideal. Therefore, assuming this statement would contradict the author's position.
Statement III: This is a factual observation, not an assumption. While it is mentioned that cricket involves two teams, the argument does not depend on this being true. It’s used as an example, not as an essential assumption.
Therefore, only Statement I is an assumption, and the correct answer is Option 1.
Was this answer helpful?
0
0